Tải bản đầy đủ (.pdf) (30 trang)

Tài liệu chuyên toán - Bất đẳng thức hiện đại - phần 8 pdf

Bạn đang xem bản rút gọn của tài liệu. Xem và tải ngay bản đầy đủ của tài liệu tại đây (277.52 KB, 30 trang )

203
Lời giải. Bất đẳng thức tương đương với
X
cy c
a + 2b
c + 2b
 3  2
X
cy c
a
b + c
 3
,


2
P
cy c
a
3
+ 3abc  3
P
cy c
a
2
b
(2a + b)(2b + c)(2c + a)


2
P
cy c
a
3

P
cy c
ab(a + b)
(a + b)(b + c)(c + a)
, 2

X
cy c
a
3
+ 3abc  3
X
cy c
a
2
b 
"
2

X
cy c
a
3

X
cy c
ab(a + b)
#

(2a + b)(2b + c)(2c + a)
(a + b)(b + c)(c + a)

Do 2
P
cy c
a
3

P
cy c
ab(a + b)  0 và
(2a+b)(2b+c)(2c+a)
(a+b)(b+c)(c+a)
 2 nên ta chỉ cần chứng minh được

2
X
cy c
a
3
+ 3abc  3
X
cy c
a
2
b  2
"

2
X
cy c
a
3

X
cy c
ab(a + b)
#
, 2
X

cy c
a
3
 2
X
cy c
ab
2
+
X
cy c
a

2
b  3abc  0
Theo bất đẳng thức AM-GM, ta có
2
X
cy c
a
3
 2
X
cy c
ab

2
 0;
X
cy c
a
2
b  3abc  0:
Bất đẳng thức được chứng minh. Đẳng thức xảy ra khi và chỉ khi a = b = c:
Bài toán 2.4 Cho các số dương a; b; c thỏa mãn (a + b+c)

1
a

+
1
b
+
1
c

= 10: Chứng
minh rằng
7 + 8
p
2  5

p
5
2

a
b
+
b
c
+
c
a


7 + 5
p
5  8
p
2
2
:
(Phạm Kim Hùng, Võ Quốc Bá Cẩn)
Lời giải. Do tính thuần nhất, không mất tính tổng quát giả sử a + b + c = 1; đặt
q = ab + bc + ca; r = abc thì ta có q = 10r: Ta có
a

b
+
b
c
+
c
a
=
ab
2
+ bc
2

+ ca
2
abc
204 CHƯƠNG 2. SÁNG TẠO BẤT ĐẲNG THỨC
Do đó, để chứng minh bất đẳng thức bên trái, ta chỉ cần xét nó trong trường hợp
c  b  a là đủ, từ đó
a
b
+
b
c
+

c
a
=
ab
2
+ bc
2
+ ca
2
abc
=
q 3r +

p
q
2
 4q
3
+ 2(9q 2)r 27r
2
2r
=
7
2
+

1
2
v
u
u
t
q
2
 4q
3
+ 2(9q 2) 
q

10
 27

q
10

2

q
10

2

=
7
2
+
1
2
s
253  40

10q +
1
q



7
2
+
1
2
q
253  80
p
10
=

7 + 8
p
2  5
p
5
2
Tiếp theo, ta sẽ chứng minh bất đẳng thức bên phải, rõ ràng ta chỉ cần xét nó trong
trường hợp a  b  c là đủ, khi đó
a
b
+
b

c
+
c
a
=
ab
2
+ bc
2
+ ca
2
abc

=
q 3r 
p
q
2
 4q
3
+ 2(9q 2)r 27r
2
2r
=
7

2

1
2
v
u
u
t
q
2
 4q
3

+ 2(9q 2) 
q
10
 27

q
10

2

q
10


2
=
7
2

1
2
s
253  40

10q +

1
q


7
2

1
2
q
253  80
p

10
=
7 + 5
p
5  8
p
2
2
:
Bất đẳng thức được chứng minh xong. Đẳng thức ở bất đẳng thức bên trái xảy
ra khi và chỉ khi c =
10

p
10+5
p
22
p
5
20
; b =
p
10
10
; a =

10
p
105
p
2+2
p
5
20
và các hoán
vị tương ứng. Đẳng thức ở bất đẳng thức bên phải xảy ra khi và chỉ khi a =
10
p

10+5
p
22
p
5
20
; b =
p
10
10
; c =
10

p
105
p
2+2
p
5
20
và các hoán vị tương ứng.
Bài toán 2.5 Cho các số dương a; b; c thỏa mãn a + b + c = 3: Chứng minh rằng
a
4
+ b

4
+ c
4
+ 4

p
ab +
p
bc +
p
ca


 15:
(Dương Đức Lâm)
Lời giải. Trước hết, ta sẽ chứng minh rằng
X
cy c
1
a + b

P
cy c
a
P

cy c
ab
+
P
cy c
a
2
P
cy c
a
2
205

,
X
cy c
c(a + b) + ab
a + b

X
cy c
a +

P
cy c

a
!
P
cy c
ab
!
2
P
cy c
a
2
,

X
cy c
ab
a + b


P
cy c
a
!
P
cy c

ab
!
2
P
cy c
a
2
Sử dụng bất đẳng thức Cauchy Schwarz, ta có
X
cy c
ab
a + b



P
cy c
ab
!
2
P
cy c
ab(a + b)
Nên ta chỉ cần chứng minh
2


X
cy c
a
2
!
X
cy c
ab
!



X
cy c
a
!"
X
cy c
ab(a + b)
#
,
X
cy c
ab(a  b)

2
 0
Trở lại bài toán của ta, sử dụng bất đẳng thức GM-HM, ta có
X
cy c
p
ab  2
X
cy c
ab
a + b
Nên ta chỉ cần chứng minh được

X
cy c
a
4
+ 8
X
cy c
ab
a + b
 15
,
X

cy c
a
4
+
8
3
X
cy c
ab(a + b + c)
a + b
 15
,

X
cy c
a
4
+
8
3
X
cy c
ab +
8
3

abc
X
cy c
1
a + b
 15
Theo trên, ta có
1
3
X
cy c
1

a + b

1
P
cy c
ab
+
1
2
P
cy c
a

2
206 CHƯƠNG 2. SÁNG TẠO BẤT ĐẲNG THỨC
Nên ta chỉ cần chứng minh được
X
cy c
a
4
+
8
3
X
cy c

ab + 8abc
0
B
@
1
P
cy c
ab
+
1
2
P

cy c
a
2
1
C
A
 15
Đặt q = ab + bc + ca; r = abc thì ta có
X
cy c
a
4

= 81 36q + 2q
2
+ 12r
Nên bất đẳng thức tương đương với
33 
50
3
q + q
2
+
12r(3 + 4q  q
2

)
q(9 2q)
 0
Nếu 9  4q thì ta có
33 
50
3
q + q
2
 0
nên bất đẳng thức đúng.
Nếu 4q  9 thì theo bất đẳng thức Schur bậc 4, ta có r 

(4q 9) (9 q)
18
nên ta chỉ cần
chứng minh
33 
50
3
q + q
2
+
2(4q 9)(9 q)(3 + 4q q
2

)
3q(9 2q)
 0
, (99 50q + 3q
2
)q(9 2q) + 2(4q  9)(9  q)(3 + 4q  q
2
)  0
, (q  3)(2q
3
+ 11q
2

 117q + 162)  0
, f(q) = 2q
3
+ 11q
2
 117q + 162  0
Dễ thấy f(q) là hàm lồi nên
f(q)  max

f(3); f

9

4

= max

36; 
729
32

< 0:
Bất đẳng thức được chứng minh xong. Đẳng thức xảy ra khi và chỉ khi a = b = c = 1:
Bài toán 2.6 Cho các số dương a; b; c thỏa mãn a  b + c: Chứng minh rằng
a

b + c
+
b
c + a
+
c
a + b
+
s
abc
(a + b)(b + c)(c + a)
 2:

(Dương Đức Lâm)
207
Lời giải. Do a  b + c nên
(a + b)(a + c)(b + c)
abc
=
b + c
bc

a + b + c +
bc
a


=
b + c
bc

2b + 2c +
bc
b + c
+
(a  b  c)(ab + ac  bc)
a(b + c)



b + c
bc

2b + 2c +
bc
b + c

=
2(b + c)
2
bc

+ 1  9
) (a + b)(a + c)(b + c)  9abc
Do đó
V T =
X
cy c
a
b + c
+
p
abc(a + b)(b + c)(c + a)
(a + b)(b + c)(c + a)

 2

X
cy c
a
b + c
+
3abc
(a + b)(b + c)(c + a)
 2
=
(a  b  c)(a + b  c)(a  b + c)

(a + b)(b + c)(c + a)
 0:
Bất đẳng thức được chứng minh xong. Đẳng thức xảy ra khi và chỉ khi a = b; c = 0
hoặc a = c; b = 0 hoặc a = 2b = 2c:
Bài toán 2.7 Cho các số không âm a; b; c, không có 2 số nào đồng thời bằng 0: Chứng
minh rằng
3
r
a
2
+ bc
b

2
+ c
2
+
3
r
b
2
+ ca
c
2
+ a

2
+
3
r
c
2
+ ab
a
2
+ b
2
 2 +

1
3
p
2
:
(Võ Quốc Bá Cẩn)
Lời giải. Không mất tính tổng quát, giả sử a  b  c; ta sẽ chứng minh
3
r
a
2
+ bc

b
2
+ c
2
+
3
r
b
2
+ ca
c
2

+ a
2
 max
(
2;
3
r
4(a
2
+ b
2
)

c
2
+ ab
)
Chú ý rằng
(a
2
+ bc)(b
2
+ ca)
(a
2

+ c
2
)(b
2
+ c
2
)

(a
2
+ c
2

)(b
2
+ c
2
)
(a
2
+ c
2
)(b
2
+ c

2
)
= 1
nên
3
r
a
2
+ bc
b
2
+ c

2
+
3
r
b
2
+ ca
c
2
+ a
2
 2

6
s
(a
2
+ bc)(b
2
+ ca)
(a
2
+ c
2
)(b

2
+ c
2
)
 2
208 CHƯƠNG 2. SÁNG TẠO BẤT ĐẲNG THỨC
Ta cần chứng minh
3
r
a
2
+ bc

b
2
+ c
2
+
3
r
b
2
+ ca
c
2

+ a
2

3
r
4(a
2
+ b
2
)
c
2

+ ab
,
a
2
+ bc
b
2
+ c
2
+
b
2

+ ca
c
2
+ a
2
+3
3
s
(a
2
+ bc)(b
2

+ ca)
(a
2
+ c
2
)(b
2
+ c
2
)

3

r
a
2
+ bc
b
2
+ c
2
+
3
r
b

2
+ ca
c
2
+ a
2
!

4(a
2
+ b
2

)
c
2
+ ab
Lại có
b
2
+ ca
b
2
+ c
2


a
2
+ bc
a
2
+ c
2
=
c(a  b)(a
2
+ b

2
+ c
2
+ ab  ac  bc)
(a
2
+ c
2
)(b
2
+ c
2

)
 0
)
b
2
+ ca
b
2
+ c
2

a

2
+ bc
a
2
+ c
2
 1
Do đó, theo bất đẳng thức AM-GM,
3
3
s
(a

2
+ bc)(b
2
+ ca)
(a
2
+ c
2
)(b
2
+ c
2

)

3
r
a
2
+ bc
b
2
+ c
2
+

3
r
b
2
+ ca
c
2
+ a
2
!
 6
s

(a
2
+ bc)(b
2
+ ca)
(a
2
+ c
2
)(b
2
+ c

2
)

6(a
2
+ bc)
a
2
+ c
2
Từ đó, ta chỉ cần chứng minh được
a

2
+ bc
b
2
+ c
2
+
b
2
+ ca
c
2

+ a
2
+
6(a
2
+ bc)
a
2
+ c
2

4(a

2
+ b
2
)
c
2
+ ab
, f(c) + g(c)  0
trong đó
f(c) = (a + 7b)c
5
+ 3(a

2
 b
2
)c
4
+ 2(a + b)(a + 3b)bc
3
 0
g(c) = (a  b)(3b
3
+ 2ab
2

+ 4a
2
b  3a
3
)c
2
+ (b
2
a
3
+ 6b
4

a + a
2
b
3
)c + ab(a  b)
4
Ta sẽ chứng minh rằng g(c)  0. Nếu 3b
3
+ 2ab
2
+ 4a
2

b  3a
3
 0, điều này là
hiển nhiên. Nếu 3b
3
+ 2ab
2
+ 4a
2
b  3a
3
 0, khi đó do g(c) là hàm lõm theo c nên

g(c)  min fg(0); g(b)g, mà
g(0) = ab(a  b)
4
 0; g(b) = b

1
4
(a  b)[(2a
2
 6ab  b
2
)

2
+ 43b
4
] + 8b
5

 0
Khẳng định được chứng minh. Trở lại bài toán của ta, có 2 trường hợp xảy ra
Nếu
a
2
+b

2
c
2
+ab
 2 ,
c
2
+ab
a
2
+b
2


1
2
, khi đó từ khẳng định trên, ta dễ dàng đi đến kết luận.
Nếu
a
2
+b
2
c
2
+ab

 2, khi đó từ khẳng định trên, ta chỉ cần chứng minh
3
r
4(a
2
+ b
2
)
c
2
+ ab
+

3
r
c
2
+ ab
a
2
+ b
2
 2 +
1
3

p
2
209
Bất đẳng thức này hiển nhiên đúng do hàm
3
p
4x +
1
x
là hàm tăng với mọi x 
3
p

2.
Bất đẳng thức được chứng minh xong. Đẳng thức xảy ra khi và chỉ khi a = b; c = 0
hoặc các hoán vị.
Bài toán 2.8 Cho a; b; c là các số không âm thỏa mãn a + b + c = 3. Chứng minh
rằng
a
2
b + b
2
c +
3
2

abc  4:
(Vasile Cirtoaje, Võ Quốc Bá Cẩn)
Lời giải. Nếu a  2b thì
(a + c)
2
b  a
2
b  b
2
c 
3
2

abc =
bc(a  2b + 2c)
2
 0
) a
2
b + b
2
c +
3
2
abc  (a + c)

2
b = 4 
a + c
2

a + c
2
 b
 4

a+c
2

+
a+c
2
+ b
3

3
= 4
Nếu 2b  a, bất đẳng thức tương đương với
f(c) = 4(a + b + c)
3
 27a

2
b  27b
2
c 
81
2
abc  0
Ta có
f
0
(c) =
3

2
[8(a + b + c)
2
 9b(3a + 2b)]
f
0
(c) = 0 , c =
3
2
p
2
p

b(3a + 2b)  a  b
Do 2b  a nên
3
2
p
2
p
b(3a + 2b)  a + b, và ta dễ dàng kiểm tra được
f(c)  f

3
2

p
2
p
b(3a + 2b)  a  b

=
27ab(a  2b)
2
(2a + b)
4

a

2
+ 5ab + 2b
2
+
q
b(3a+2b)
3
2

 0:
Bất đẳng thức được chứng minh. Đẳng thức xảy ra khi và chỉ khi a = 2; b = 1; c = 0
hoặc a = 0; b = 2; c = 1.

Bài toán 2.9 Cho a; b; c là các số không âm, không có 2 số nào đồng thời bằng 0.
Chứng minh rằng
1
a
2
+ ab + b
2
+
1
b
2
+ bc + c

2
+
1
c
2
+ ca + a
2

9
(a + b + c)
2
:

(Vasile Cirtoaje)
210 CHƯƠNG 2. SÁNG TẠO BẤT ĐẲNG THỨC
Lời giải. Bất đẳng thức được viết lại như sau
X
cy c

(a + b + c)
2
a
2
+ ab + b
2

 1

 6
,
X
cy c
c(a + b + c) + ab + bc + ca
a
2
+ ab + b
2
 6

,

X
cy c
a
!
X
cy c
c
a
2
+ ab + b

2
!
+

X
cy c
ab
!
X
cy c
1
a

2
+ ab + b
2
!
 6
Sử dụng bất đẳng thức Cauchy Schwarz, ta có
X
cy c
c
a
2
+ ab + b

2


P
cy c
c
!
2
P
cy c
c(a
2

+ ab + b
2
)
=
P
cy c
a
P
cy c
ab
X
cy c

1
a
2
+ ab + b
2


P
cy c
c
!
2

P
cy c
c
2
(a
2
+ ab + b
2
)
=

P

cy c
a
!
2
2

P
cy c
ab
!
2
 3abc

P
cy c
a
Nên ta chỉ cần chứng minh được

P
cy c
a
!
2
P
cy c

ab
+

P
cy c
a
!
2

P
cy c
ab

!
2

P
cy c
ab
!
2
 3abc
P
cy c
a

 6
Do tính thuần nhất, ta có thể giả sử a + b + c = 1. Đặt q =
P
cy c
ab; r = abc, bất đẳng
thức trở thành
1
q
+
q
2q
2

 3r
 6
Sử dụng bất đẳng thức Schur bậc 4, ta có r 
(4q 1) (1 q)
6
, nên
1
q
+
q
2q
2

 3r
 6 
1
q
+
q
2q
2

(4q 1) (1 q)
2
 6 =

(1  3q)(4q  1)
2
q(8q
2
 5q + 1)
 0:
Bất đẳng thức được chứng minh. Đẳng thức xảy ra khi và chỉ khi a = b = c:
211
Bài toán 2.10 Cho các số không âm a; b; c; không có 2 số nào đồng thời bằng 0:
Chứng minh rằng
a
4

a
2
+ ab + b
2
+
b
4
b
2
+ bc + c
2
+

c
4
c
2
+ ca + a
2

a
3
+ b
3
+ c

3
a + b + c
:
(Phan Thành Việt)
Lời giải. Ta có
a
3
+ b
3
+ c
3
a + b + c

= a
2
+ b
2
+ c
2
 ab  bc  ca +
3abc
a + b + c
Nên bất đẳng thức đã cho tương đương với
X
cy c


a
4
a
2
+ ab + b
2
+ ab  a
2


3abc

a + b + c
,
X
cy c
ab
3
a
2
+ ab + b
2

3abc

a + b + c
Sử dụng bất đẳng thức Cauchy Schwarz, ta có

X
cy c
ab
3
a
2
+ ab + b
2
!

X
cy c
a
2
+ ab + b
2
ab
!


X
cy c

a
!
2
Nên ta chỉ cần chứng minh

X
cy c
a
!
2

3abc

a + b + c
X
cy c
a
2
+ ab + b
2
ab
,

X
cy c

a
!
3
 3
X
cy c
c(a
2
+ ab + b
2
)
,


X
cy c
a
!
X
cy c
a
2

X
cy c

ab
!
 0:
Điều này hiển nhiên đúng. Đẳng thức xảy ra khi và chỉ khi a = b = c hoặc c = 0;
a
b
! 0
hoặc các hoán vị tương ứng.
Bài toán 2.11 Cho a; b; c; d là các số không âm thỏa mãn a + b + c + d = 1: Chứng
minh rằng
a
p

a + b
+
b
p
b + c
+
c
p
c + d
+
d
p

d + a

3
2
:
(Mircea Lascu)
212 CHƯƠNG 2. SÁNG TẠO BẤT ĐẲNG THỨC
Lời giải. Không mất tính tổng quát, ta có thể giả s ử a + c  b + d ) x = a + c 
1
2
:
Sử dụng bất đẳng thức Jack Garfunkel, ta có

a
p
a + b
+
b
p
b + c
+
c
p
c + a


5
4
p
a + b + c =
5
4
p
1  d
)
a
p
a + b

+
b
p
b + c

5
4
p
1  d 
c
p
c + a

c
p
c + d
+
d
p
d + a
+
a
p
a + c


5
4
p
a + c + d =
5
4
p
1  b
)
c
p
c + d

+
d
p
d + a

5
4
p
1  b 
a
p
a + c

Suy ra
X
cy c
a
p
a + b

5
4

p
1  b +

p
1  d


p
a + c 
5
4
p
2(2  b  d) 
p
c + a

=
5
4
p
2(x + 1) 
p
x =
(
p
x  1) (17
p
x  7)

2
p
2

5
p
x + 1 +
p
2 (2
p
x + 3)


+
3
2

3
2
:
Bất đẳng thức được chứng minh xong. Đẳng thức không xảy ra.
Bài toán 2.12 Cho a; b; c; d là các số không âm, không có 3 số nào đồng thời bằng
0: Chứng minh rằng
a
p

a + b + c
+
b
p
b + c + d
+
c
p
c + d + a
+
d
p

d + a + b

5
4
p
a + b + c + d:
Lời giải. Giả sử d = minfa; b; c; dg và đặt x = a + c; khi đó ta dễ thấy
a
p
a + b + c
+
d

p
d + a + b

a
p
a + b + d
+
d
p
d + a + b
=
x

p
x + b
b
p
b + c + d

b
p
b + c
Nên ta chỉ cần chứng minh
x
p

x + b
+
b
p
b + c
+
c
p
c + d + a

5
4

p
a + b + c + d
,
x
p
x + b
+
b
p
b + c
+
c

p
c + x

5
4
p
x + b + d
Đây chính là bất đẳng thức Jack Garfunkel nên bất đẳng thức đã cho được chứng
minh. Đẳng thức xảy ra khi và chỉ khi
a
3
=

b
1
=
c
0
=
d
0
hoặc các hoán vị tương ứng.
213
Bài toán 2.13 Cho a; b; c là các số dương. Chứng minh rằng
2(a + b + c)(bc + ca + ab)

a
2
+ b
2
+ c
2
+ ab + bc + ca
 ((2a + b)(2b + c)(2c + a))
1
3
:
(Sung Yoon Kim)

Lời giải 1. Sử dụng bất đẳng thức Cauchy Schwarz, ta có a
2
b + b
2
c + c
2
a 
(ab+bc+ca)
2
a+b+c
. Suy ra
) (2a + b)(2b + c)(2c + a)

= 2(a + b + c)(ab + bc + ca) + 3abc + 2(a
2
b + b
2
c + c
2
a)
 2(a + b + c)(ab + bc + ca) + 3abc +
2(ab + bc + ca)
2
a + b + c
Ta cần chứng minh

2(a + b + c)(bc + ca + ab)
a
2
+ b
2
+ c
2
+ ab + bc + ca


2(a + b + c)(ab + bc + ca) + 3abc +
2(ab + bc + ca)

2
a + b + c

1
3
Do tính thuần nhất, ta có thể chuẩn hóa cho a+b+c = 1: Đặt q = ab+bc+ca; r = abc;
khi đó theo bất đẳng thức Schur bậc 3, ta được r  max

0;
4q 1
9


. Bất đẳng thứ c
trở thành
(3r + 2q
2
+ 2q)
1
3

2q
1  q
Nếu 1  4q; thì
(3r + 2q

2
+ 2q)
1
3
 (2q
2
+ 2q)
1
3

2q
2

+ 2q 
8q
3
(1  q)
3
=
2q(1 2q  4q
2
+ 2q
3
 q
4

)
(1  q)
3

2q(1 2q  4q
2
)
(1  q)
3

2q(1 3q)
(1  q)

3
 0
Nếu 4q  1; thì
(3r + 2q
2
+ 2q)
1
3


2q
2

+ 2q +
4q 1
3

1
3
=

6q
2
+ 10q 1
3


1
3

6q
2
+ 10q 1
3

8q
3
(1  q)

3
=
(1  3q)(2q
4
 2q
3
+ 3q
2
+ 10q 1)
(1  q)
3
 0:

214 CHƯƠNG 2. SÁNG TẠO BẤT ĐẲNG THỨC
Đẳng thức xảy ra khi và chỉ khi a = b = c:
Lời giải 2. Sử dụng bất đẳng thức Holder, ta có
[(a + b + c)(bc + ca + ab)]
3
= [(a + b)(b + c)(c + a) + abc]
3
 [(a + b)
3
+ a
3
][(b + c)

3
+ b
3
][(c + a)
3
+ c
3
]
= (2a + b)(2b + c)(2c + a)(a
2
+ ab + b
2

)(b
2
+ bc + c
2
)(c
2
+ ca + a
2
)
Ta cần chứng minh
8(a
2

+ ab + b
2
)(b
2
+ bc + c
2
)(c
2
+ ca + a
2
)  (a
2

+ b
2
+ c
2
+ bc + ca + ab)
3
Bất đẳng thức này có thể được chứng minh bằng phép khai triển.
Lời giải 3. Ta sẽ chứng minh kết quả mạnh hơn là
a + 2b
a + 2c
+
b + 2c

b + 2a
+
c + 2a
c + 2b

3(a
2
+ b
2
+ c
2
)

ab + bc + ca
Cộng 3 vào hai vế, ta viết được bất đẳng thức trên ở dạng
2(a + b + c)

1
a + 2c
+
1
b + 2a
+
1
c + 2b



3(a
2
+ b
2
+ c
2
)
ab + bc + ca
+ 3
Từ đây, sử dụng bất đẳng thức AM-GM, ta có thể suy ra bất đẳng thức ban đầu của

bài toán.
Do (a + b + c)
2
= (a + 2b)(a + 2c) + (b c)
2
nên
a + b + c
a + 2c
=
a + 2b
a + b + c
+

(b  c)
2
(a + 2c)(a + b + c)
)
X
cy c
a + b + c
a + 2c
= 3 +
X
cy c
(b  c)

2
(a + 2c)(a + b + c)
Do đó, bất đẳng thức trên tương đương với
6 + 2
X
cy c
(b  c)
2
(a + 2c)(a + b + c)

3(a
2

+ b
2
+ c
2
)
ab + bc + ca
+ 3
, x(b c)
2
+ y(c a)
2
+ z(a  b)

2
 0
với x =
3(a+b+c)
4(ab+bc+ca)

1
a+2c
và y; z tương tự.
Không mất tính tổng quát, giả sử b là số hạng nằm giữa a và c; tức là (ba)(bc)  0:
215
Từ đó, ta có (a  b)(a  c)  0 và (c  a)(c  b)  0: Chú ý rằng (a + 2b)(a + 2c) =

3(ab + bc + ca) + (a  b)(a  c), ta có
3(ab + bc + ca)
a + 2c
= a + 2b 
(a  b)(a  c)
a + 2c
 a + 2b  2(a + b + c) 
9(a + b + c)
4
nên x  0: Tương tự, ta cũng có z  0:
Nếu a  b  c; khi đó dễ thấy y  0 do
1

b + 2a

1
a + b + c

a + b + c
3(ab + bc + ca)

3(a + b + c)
4(ab + bc + ca)
nên bất đẳng thức đúng.
Nếu c  b  a và nếu y  0 nên bất đẳng thức đúng. Giả sử y  0; khi đó ta có

x + 2y =
9(a + b + c)
4(ab + bc + ca)

1
a + 2c

2
b + 2a
 0
do
1

a + 2c
+
2
b + 2a
=
4a + b + 4c
(a + 2c)(b + 2a)

2(a + b + c)
ab + bc + ca

9(a + b + c)

4(ab + bc + ca)

z + 2y =
9(a + b + c)
4(ab + bc + ca)

1
c + 2b

2
b + 2a
 0

do
1
c + 2b
+
2
b + 2a
=
2a + 5b + 2c
(c + 2b)(b + 2a)

2(a + b + c)
ab + bc + ca


9(a + b + c)
4(ab + bc + ca)
Từ đây, với chú ý rằng (a  c)
2
 2(a b)
2
+ 2(b  c)
2
; ta được
x(b  c)
2

+ y(c a)
2
+ z(a  b)
2
 (x + 2y)(b  c)
2
+ (z + 2y)(a  b)
2
 0:
Bất đẳng thức được chứng minh.
Bài toán 2.14 Cho các số dương a; b; c: Chứng minh rằng
3(a

2
+ b
2
+ c
2
)
a + b + c

a(a + b)
a + c
+
b(b + c)

b + a
+
c(c + a)
c + b
:
(Phạm Hữu Đức)
Lời giải 1. Chú ý rằng
X
cy c
a(a + b)
a + c
=

X
cy c
a(a + b + c)
a + c

X
cy c
ac
a + c
=

X

cy c
a
!
X
cy c
a
a + c
!

X
cy c
ab

a + b
=

X
cy c
a
!
3 
X
cy c
c
a + c

!

X
cy c
ab
a + b
216 CHƯƠNG 2. SÁNG TẠO BẤT ĐẲNG THỨC
Ta có thể viết lại bất đẳng thức n hư sau
3
P
cy c
a

2
P
cy c
a
+
X
cy c
ab
a + b
+

X

cy c
a
!
X
cy c
c
a + c
!
 3
X
cy c
a

Sử dụng bất đẳng thức Cauchy Schwarz, ta có
X
cy c
ab
a + b


P
cy c
ab
!
2

P
cy c
ab(a + b)
=

P
cy c
ab
!
2

P

cy c
a
!
P
cy c
ab
!
 3abc

X
cy c
c

a + c


P
cy c
c
!
2
P
cy c
c(a + c)
=


P
cy c
a
!
2

P
cy c
a
!
2


P
cy c
ab
Ta cần chứng minh
3
P
cy c
a
2
P
cy c

a
+

P
cy c
ab
!
2

P
cy c
a

!
P
cy c
ab
!
 3abc
+

P
cy c
a
!

3

P
cy c
a
!
2

P
cy c
ab
 3

X
cy c
a
Do tính thuần nhất, ta có thể chuẩn hóa cho a+b+c = 1: Đặt q = ab+bc+ca; r = abc;
khi đó theo bất đẳng thức Schur bậc 3, ta có r 
4q 1
9
. Bất đẳng thức trở thành
3(1  2q) +
q
2
q 3r

+
1
1  q
 3
,
q
2
q 3r
+
1
1  q
 6q  0

Ta có
q
2
q 3r
+
1
1  q
 6q 
q
2
q 
4q 1

3
+
1
1  q
 6q =
(1  3q)
2
1  q
 0:
Bất đẳng thức được chứng minh xong. Đẳng thức xảy ra khi và chỉ khi a = b = c:
217
Lời giải 2. Bất đẳng thức đã cho tương đương với

X
cy c
a(a + b)(a + b + c)
a + c
 3(a
2
+ b
2
+ c
2
)
,

X
cy c
ab(a + b)
a + c
 2
X
cy c
a
2

X
cy c

ab
Gọi x; y; z là một hoán vị của a; b; c sao cho x  y  z; khi đó với chú ý rằng
xy(x + y)  xz(x + z)  yz(y + z)
Sử dụng bất đẳng thức sắp xếp lại, ta có
X
cy c
ab(a + b)
a + c

xy(x + y)
y + z
+

yz(y + z)
y + x
+
zx(z + x)
z + x
=
xy(x + y)
y + z
+
yz(y + z)
y + x
+ xz

Bây giờ, ta thấy
xy(x + y)
y + z
+
yz(y + z)
y + x
 (xy + yz) =
xy(x z)
y + z
+
yz(z  x)
x + y

=
y(x z)
2
(x + y + z)
(x + y)(y + z)
 (x z)
2
Từ đây, ta có
X
cy c
ab(a + b)
a + c

 (x  z)
2
+ (xy + yz + xz )
 2(x
2
+ y
2
+ z
2
)  (xy + yz + zx):
Bất đẳng thức được chứng minh.
Bài toán 2.15 Cho các số không âm a; b; c; không có 2 số nào cùng bằng 0: Chứng

minh rằng
r
a
b + 3c
+
r
b
c + 3a
+
r
c
a + 3b


3
2
:
(Vasile Cirtoaje)
Lời giải. Đặt
c
a+3b
=
z
2
4

;
b
c+3a
=
y
2
4
;
a
b+3c
=
x

2
4
; với x; y; z là các số không âm. Khi
đó, ta dễ dàng kiểm tra được đẳng thứ c sau
16 = 7x
2
y
2
z
2
+ 3(x
2

y
2
+ x
2
z
2
+ y
2
z
2
)
218 CHƯƠNG 2. SÁNG TẠO BẤT ĐẲNG THỨC

và ta cần chứng minh
x + y + z  3
Giả sử x + y +z < 3; khi đó tồn tại k > 1 sao cho kx +ky +kz = 3. Đặt kx = u; ky =
v; kz = w thì u + v + w = 3 và
16 =
7u
2
v
2
w
2
k

6
+
3(u
2
v
2
+ u
2
w
2
+ v
2

w
2
)
k
4
< 7u
2
v
2
w
2
+ 3(u

2
v
2
+ u
2
w
2
+ v
2
w
2
)

Nhưng ta dễ dàng chứng minh được
16  7u
2
v
2
w
2
+ 3(u
2
v
2
+ u

2
w
2
+ v
2
w
2
)
với mọi u; v; w  0 thỏa mãn u + v + w = 3; điều này dẫn đến mâu thuẫn nên bất
đẳng thức cần chứng minh đúng.
Đẳng thức xảy ra khi và chỉ khi a = b = c:
Bài toán 2.16 Cho các số dương a; b; c: Chứng minh rằng

3
p
(a + b)
2
(b + c)
2
(c + a)
2

3
p
(a  b)

2
(b  c)
2
(c  a)
2
+ 4
3
p
a
2
b
2

c
2
:
(Sung Yoon Kim)
Lời giải. Sử dụng bất đẳng thức AM-GM, ta có
3
s
(a  b)
2
(b  c)
2
(c  a)

2
(a + b)
2
(b + c)
2
(a + c)
2
+ 4
3
s
a
2

b
2
c
2
(a + b)
2
(b + c)
2
(a + c)
2
=
3

s
(a  b)
2
(a + b)
2

(b  c)
2
(b + c)
2

(c  a)

2
(c + a)
2
+ 4
3
s
ab
(a + b)
2

bc
(b + c)

2

ca
(c + a)
2

1
3
X
cy c
(a  b)
2

(a + b)
2
+
4
3
X
cy c
ab
(a + b)
2
=
1

3
X
cy c

(a  b)
2
(a + b)
2
+
4ab
(a + b)
2


= 1:
Bất đẳng thức được chứng minh. Đẳng thức xảy ra khi và chỉ khi a = b = c:
Bài toán 2.17 Cho a; b; c là các số dương thỏa mãn a
2
b
2
+ b
2
c
2
+ c

2
a
2
= 3: Chứng
minh rằng
r
a + bc
2
2
+
r
b + ca

2
2
+
r
c + ab
2
2

3
abc
:
(Sung Yoon Kim)

219
Lời giải. Sử dụng bất đẳng thứ AM-GM, ta có
X
cy c
r
a + bc
2
2
=
X
cy c
r

1
b

ab + b
2
c
2
2

1
2
X

cy c

1
b
+
ab + b
2
c
2
2

=

1
4

2
X
cy c
1
a
+
X
cy c
ab + 3

!

X
cy c
ab 
1
2
X
cy c
(a
2
b

2
+ 1) = 3
)
X
cy c
1
a
=
P
cy c
ab
abc


3
abc
Do đó
X
cy c
r
a + bc
2
2

1

4

6
abc
+ 6

=
3
2abc
+
3
2


3
abc
:
Bất đẳng thức được chứng minh. Đẳng thức xảy ra khi và chỉ khi a = b = c = 1:
Bài toán 2.18 Cho k > 0 là một hằng số cho trước. Tìm hằng số  lớn nhất sao
cho với mọi a  b  c  d  0 thỏa a + b + c + d = 1; bất đẳng thức sau đúng
a  b
k + a + b
+
b  c
k + b + c

+
c  d
k + c + d
+
d  a
k + d + a
 (a b)(b c)(c d):
(Shalex)
Lời giải. Cho a =
1
4
+ 3; b =

1
4
+ ; c =
1
4
; d =
1
4
3 với  ! 0
+
, bất đẳng thức
trở thành

2
3

64
(2k + 1)
3
 64(2k + 1)
2
 

 0
Cho  ! 0

+
, ta được
 
64
(2k + 1)
3
220 CHƯƠNG 2. SÁNG TẠO BẤT ĐẲNG THỨC
Mặt khác, đặt x = a  b; y = b c; z = c  d ) x; y; z  0, ta có
a  b
k + a + b
+
b  c

k + b + c
+
c  d
k + c + d
+
d  a
k + d + a
=

a  b
k + a + b
+

c  d
k + c + d

+

b  c
k + b + c
+
d  a
k + d + a

= [k(a b + c d) + (a

2
 b
2
+ c
2
 d
2
)]


1
(k + a + b)(k + c + d)


1
(k + b + c)(k + c + d)

=
[(k + a + b)x + (k + c + d)z](x + y)(y + z)
(k + a + b)(k + b + c)(k + c + d)(k + d + a)

8xyz
p
(k + a + b)(k + c + d)
(k + a + b)(k + b + c)(k + c + d)(k + d + a)

=
8xyz
p
(k + a + b)(k + c + d)(k + b + c)
2
(k + d + a)
2

64
(2k + 1)
3
xyz:

với bất đẳng thức cuối đúng theo bất đẳng thức AM-GM và giả thiết a+b + c+d = 1:
Từ đó, ta đi đến kết luận

max
=
64
(2k + 1)
3
:
Bài toán 2.19 Cho x
1
; x

2
; :::; x
n
là các số dương thỏa mãn
n
P
i=1
x
i
= 1: Chứng minh
rằng
n

X
i=1
q
x
2
i
+ x
2
i+1
 2 
1
p

2
2
+
n
P
i=1
x
2
i
x
i+1
:

Lời giải. Bất đẳng thức đã cho tương đương
n
X
i=1

x
i
+ x
i+1

q
x

2
i
+ x
2
i+1


1
p
2
2
+

n
P
i=1
x
2
i
x
i+1
,
n
X
i=1

x
2
i
x
2
i
x
i+1
+ x
i
+
x

i
x
i+1
q
x
2
i
+ x
2
i+1

1

p
2 + 2
n
P
i=1
x
2
i
x
i+1
221
Sử dụng bất đẳng thức Cauchy Schwarz, ta có

0
@
n
X
i=1
x
2
i
x
2
i
x

i+1
+ x
i
+
x
i
x
i+1
q
x
2
i

+ x
2
i+1
1
A
"
n
X
i=1

x
2

i
x
i+1
+ x
i
+
x
i
x
i+1
q
x

2
i
+ x
2
i+1

#


n
X
i=1

x
i
!
2
= 1
Ta cần chứng minh
n
X
i=1

x
i

x
i+1
q
x
2
i
+ x
2
i+1

x
2

i
x
i+1


p
2  1
,
n
X
i=1
x

i
x
i+1
q
x
2
i
+ x
2
i+1
+ x
i


p
2  1
Do
q
x
2
i
+ x
2
i+1


x
i
+ x
i+1
p
2
Nên ta chỉ cần chứng minh được
n
X
i=1
x
i

x
i+1

1 +
p
2

x
i
+ x
i+1
 1 

p
2
2
,
n
X
i=1
x
i
x
i+1


1 +
p
2

x
i
+ x
i+1

n
X
i=1


3  2
p
2
2
x
i
+
p
2  1
2
x

i+1
!
,
n
X
i=1

p
2  1

(x
i

 x
i+1
)
2
2

1 +
p
2

x
i

+ x
i+1

 0:
Bất đẳng thức được chứng minh xong. Đẳng th ức xảy ra khi và chỉ khi x
1
= x
2
=
 = x
n
=

1
n
:
Bài toán 2.20 Cho các số thực a; b; c; d: Chứng minh rằng
5
2
X
cy c
a
4
+
X

sy m
a
2
b
2


X
cy c
a
3
!

X
cy c
a
!
+ (a  b)(b  c)(c  d)(d  a):
(Phạm Minh Khoa)
222 CHƯƠNG 2. SÁNG TẠO BẤT ĐẲNG THỨC
Lời giải. Do
5
2
X
cy c

a
4
+
X
sy m
a
2
b
2


X

cy c
a
2
!
2
=
1
2
X
sy m
(a
2

 b
2
)
2

X
cy c
a
3
!
X
cy c

a
!


X
cy c
a
2
!
2
=
X

sy m
ab(a  b)
2
Nên ta có thể viết lại bất đẳng thức như sau
1
2
X
sy m
(a
2
 b
2

)
2

X
sy m
ab(a  b)
2
+ (a  b)(b  c)(c  d)(d  a)
,
1
2
X

sy m
(a  b)
2
(a
2
+ b
2
)  (a b)(b c)(c d)(d a)
Chú ý rằng a
2
+ b
2


1
2
(a  b)
2
, nên
1
2
X
sy m
(a  b)
2

(a
2
+ b
2
) 
1
4
X
sy m
(a  b)
4


1
4
[(a  b)
4
+ (b  c)
4
+ (c  d)
4
+ (d  a)
4
]
 j(a b)(b c)(c d)(d a)j

 (a b)(b c)(c d)(d a):
Bất đẳng thức được chứng minh xong.
Bài toán 2.21 Cho các số dương a; b; c thỏa a
2
+ b
2
+ c
2
= 3: Chứng minh rằng
r
a
2

a
2
+ b + c
+
r
b
2
b
2
+ c + a
+
r

c
2
c
2
+ a + b

p
3:
Lời giải. Theo bất đẳng thức Cauchy Schwarz, ta có a + b + c  3. Từ đó

X
cy c

r
a
2
a
2
+ b + c
!
2


X
cy c

a
!
X
cy c
a
a
2
+ b + c
!


X

cy c
a
!
X
cy c
a
a
2
+
1
3
(b + c)(a + b + c)

!
Ta cần chứng minh

X
cy c
a
!
X
cy c
a
a
2

+
1
3
(b + c)(a + b + c)
!
 3
223
Bất đẳng thức này thuần nhất nên ta có thể bỏ qua giả thiết a
2
+ b
2
+ c

2
= 3 và
chuẩn hóa cho a + b + c = 1; khi đó bất đẳng thức trở thành
X
cy c
3a
3a
2
 a + 1
 3
Ta có
X

cy c
3a
3a
2
 a + 1
 3 =
X
cy c

3a
3a
2

 a + 1
 2a 
1
3

= 
X
cy c
(2a + 1)(3a  1)
2
3(3a
2

 a + 1)
 0:
Bất đẳng thức được chứng minh. Đẳng thức xảy ra khi và chỉ khi a = b = c = 1:
Bài toán 2.22 Cho a; b; c là các số dương. Chứng minh rằng

a
b + c
2
+
b
c + a
2

+
c
a + b
2

2

1
4

a
c

2
+
b
a
2
+
c
b
2

a
b

+
b
c
+
c
a

:
Lời giải. Sử dụng bất đẳng thức AM-GM, ta có
a
b + c
2

+
b
c + a
2
+
c
a + b
2


a
2

p
bc
+
b
2
p
ca
+
c
2
p
ab


=
1
2

p
a
p
b

p
a

c
+
p
b
p
c

p
b
a
+
p

c
p
a

p
c
b
!

1
2
s


a
c
2
+
b
a
2
+
c
b
2


a
b
+
b
c
+
c
a

:
Bất đẳng thức được chứng minh. Đẳng thức xảy ra khi và chỉ khi a = b = c = 1:

Bài toán 2.23 Cho các số dương a; b; c: Chứng minh rằng
a
b
+
b
c
+
c
a

r
a

2
+ c
2
b
2
+ c
2
+
r
b
2
+ a

2
c
2
+ a
2
+
r
c
2
+ b
2
a

2
+ b
2
:
(Võ Quốc Bá Cẩn)
Lời giải 1. Bình phương 2 vế, ta có thể viết lại bất đẳng thức như sau
X
cy c

a
2
b

2
+
2b
a

a
2
+ c
2
b
2
+ c

2
 2
r
b
2
+ c
2
a
2
+ c
2
!

 0
224 CHƯƠNG 2. SÁNG TẠO BẤT ĐẲNG THỨC
Suy ra, ta chỉ cần chứng minh
a
2
b
2
+
2b
a

a

2
+ c
2
b
2
+ c
2
 2
r
b
2
+ c

2
a
2
+ c
2
 0
,
a
3
c
2
+ 2b

5
+ 2b
3
c
2
 ab
2
c
2
ab
2
(b

2
+ c
2
)
 2
r
b
2
+ c
2
a
2

+ c
2
Sử dụng bất đẳng thức AM-GM, ta có
a
3
c
2
+ 2b
3
c
2
 3ab

2
c
2
) a
3
c
2
+ 2b
5
+ 2b
3
c

2
 ab
2
c
2
 2b
2
(b
3
+ ac
2
)

Ta cần phải chứng minh
b
3
+ ac
2
a(b
2
+ c
2
)

r

b
2
+ c
2
a
2
+ c
2
Nhưng bất đẳng thức này hiển nhiên vì
(b
3
+ ac

2
)
2
(a
2
+ c
2
)  a
2
(b
2
+ c

2
)
3
= c
2
(a  b)
2
(a
2
c
2
+ 2b

3
a + 2c
2
ab + b
4
)  0:
Đẳng thức xảy ra khi và chỉ khi a = b = c:
Lời giải 2. Trước hết, ta sẽ chứng minh bất đẳng thức sau
a
b
+
b

a

r
a
2
+ c
2
b
2
+ c
2
+

r
b
2
+ c
2
a
2
+ c
2
,
a
b

+
b
a
 2 
r
a
2
+ c
2
b
2
+ c

2
+
r
b
2
+ c
2
a
2
+ c
2
 2

,
(a  b)
2
ab


p
a
2
+ c
2


p
b
2
+ c
2

2
p
(a
2
+ c
2

)(b
2
+ c
2
)
,
(a  b)
2
ab

(a
2

 b
2
)
2
p
(a
2
+ c
2
)(b
2
+ c

2
)

p
a
2
+ c
2
+
p
b
2

+ c
2

2
,
p
(a
2
+ c
2
)(b
2

+ c
2
)

p
a
2
+ c
2
+
p
b

2
+ c
2

2
 ab(a + b)
2
Bất đẳng thức cuối hiển nhiên đúng vì
p
a
2
+ c

2
+
p
b
2
+ c
2
 a + b; (a
2
+ c
2
)(b

2
+ c
2
)  a
2
b
2
Từ đó, ta có
X
cy c
a
b

+
X
cy c
b
a

X
cy c
r
a
2
+ c

2
b
2
+ c
2
+
X
cy c
r
b
2
+ c

2
a
2
+ c
2
225
Giả sử
P
cy c
a
b
<

P
cy c
q
a
2
+c
2
b
2
+c
2
, khi đó ta có

P
cy c
b
a
>
P
cy c
q
b
2
+c
2

a
2
+c
2
. Nhưng
X
cy c
a
b
<
X
cy c

r
a
2
+ c
2
b
2
+ c
2
,
X
cy c

a
2
b
2
+ 2
X
cy c
b
a
<
X
cy c

a
2
+ c
2
b
2
+ c
2
+ 2
X
cy c
r

b
2
+ c
2
a
2
+ c
2
Do
P
cy c
b

a
>
P
cy c
q
b
2
+c
2
a
2
+c

2
nên
X
cy c
a
2
b
2
<
X
cy c
a

2
+ c
2
b
2
+ c
2
điều này mâu thuẫn vì
X
cy c
a
2

b
2

X
cy c
a
2
+ c
2
b
2
+ c

2
Thật vậy, giả sử c = minfa; b; cg, thì bất đẳng thức này tương đương với

1
a
2
b
2

1
(a
2

+ c
2
)(b
2
+ c
2
)

(a
2
 b
2

)
2
+

1
a
2
c
2

1
(a

2
+ b
2
)(a
2
+ c
2
)

(a
2
 c

2
)(b
2
 c
2
)  0
hiển nhiên đúng. Vậy nên ta phải có
X
cy c
a
b


X
cy c
r
a
2
+ c
2
b
2
+ c
2
:

Bài toán 2.24 Cho các số không âm a; b; c thỏa mãn a +b +c = 1: Chứng minh rằng
p
a + (b  c)
2
+
p
b + (c  a)
2
+
p
c + (a  b)
2


p
3:
(Phan Thành Nam)
Lời giải. Bình phương 2 vế, bất đẳng thức đã cho có thể được viết lại như sau
X
cy c
(b  c)
2
+ 2
X
cy c

p
[a + (b  c)
2
][b + (a  c)
2
]  2
226 CHƯƠNG 2. SÁNG TẠO BẤT ĐẲNG THỨC
Sử dụng bất đẳng thức Cauchy Schwarz, ta có
X
cy c
p
[a + (b  c)

2
][b + (a  c)
2
] 
X
cy c
p
ab +
X
cy c
j(a  c)(b  c)j
Nên ta chỉ cần chứng minh được

X
cy c
(b  c)
2
+ 2
X
cy c
p
ab + 2
X
cy c
j(a  c)(b  c)j  2

Không mất tính tổng quát, giả sử a  b  c; khi đó
X
cy c
(b  c)
2
+ 2
X
cy c
p
ab + 2
X
cy c

j(a  c)(b  c)j  2
= 4(a  c)
2
 2

1 
X
cy c
p
ab
!
= 4(a  c)

2


p
a 
p
c

2




p
a 
p
b

2
+

p
b 
p
c


2

 4(a  c)
2
 2

p
a 
p
c


2
= 2

p
a 
p
c

2
h
2


p
a +
p
c

2
 1
i
 2

p
a 

p
c

2
[2(a + c)  1] = 2

p
a 
p
c

2

(a  b + c)  0:
Bất đẳng thức được chứng minh xong. Đẳng thức xảy ra khi và chỉ khi a = b = c =
1
3
hoặc a = b =
1
2
; c = 0 hoặc các hoán vị.
Bài toán 2.25 Cho các số dương a; b; c thỏa a + b + c = 3: Chứng minh rằng
a
2b + 1
+

b
2c + 1
+
c
2a + 1

1
abc
:
(Võ Quốc Bá Cẩn)
Lời giải. Bất đẳng thức đã cho tương đương với


3 
a
2b + 1

+

3 
b
2c + 1

+


3 
c
2a + 1

+
1
abc
 9
,
7b + c
2b + 1
+

7c + a
2c + 1
+
7a + b
2a + 1
+
1
abc
 9
, 2

c  1

2b + 1
+
a  1
2c + 1
+
b  1
2a + 1

+
2
abc
+ 3  5


1
2a + 1
+
1
2b + 1
+
1
2c + 1

Nên để chứng minh bất đẳng thức đã cho, ta chỉ cần chứng minh 2 bất đẳng thức sau
c  1

2b + 1
+
a  1
2c + 1
+
b  1
2a + 1
 0
227
2
abc
+ 3  5


1
2a + 1
+
1
2b + 1
+
1
2c + 1

a) Trước hết, ta sẽ chứng minh
c  1

2b + 1
+
a  1
2c + 1
+
b  1
2a + 1
 0
, 2(a
2
+ b
2

+ c
2
 ab  bc  ca) + 4(a
2
b + b
2
c + c
2
a)  12
, a
3
+ b

3
+ c
3
+ 12(a
2
b + b
2
c + c
2
a)  6(ab
2
+ bc

2
+ ca
2
) + 21abc
, a
3
+ b
3
+ c
3
+ 3(a
2

b + b
2
c + c
2
a + ab
2
+ bc
2
+ ca
2
)  21abc
 6(ab

2
+ bc
2
+ ca
2
 a
2
b  b
2
c  c
2
a)

, (a + b + 7c)(a  b)
2
+ (b + c + 7a)(b  c)
2
+ (c + a + 7b)(c  a)
2
 18(a  b)(b  c)(c  a)
Từ đây, giả sử a = min fa; b; cg và đặt b = a + x; c = a + y (x; y  0); ta có thể dễ
dàng biến đổi bất đẳng thức về dạng tương đương là
9(x
2
 xy + y

2
)a + x
3
+ 3x
2
y + y(3x  y)
2
 0
Nên bất đẳng thức hiển nhiên đúng.
b) Tiếp theo, ta sẽ chứng minh
2
abc

+ 3  5

1
2a + 1
+
1
2b + 1
+
1
2c + 1

Đặt q = ab + bc + ca; r = abc thì ta có

1
2a + 1
+
1
2b + 1
+
1
2c + 1
=
4q + 15
8r + 4q + 7
Nên bất đẳng thức tương đương với

3r + 2
r

5(4q + 15)
8r + 4q + 7
,
2(1  r)(4q + 7  12r)
r(8r + 4q + 7)
 0:
Bất đẳng thức cuối hiển nhiên đúng vì 3  q  3r: Vậy ta có đpcm. Đẳng thức xảy
ra khi và chỉ khi a = b = c = 1:

×